LSAT and Law School Admissions Forum

Get expert LSAT preparation and law school admissions advice from PowerScore Test Preparation.

 Administrator
PowerScore Staff
  • PowerScore Staff
  • Posts: 8917
  • Joined: Feb 02, 2011
|
#41385
Please post your questions below!
 g_lawyered
  • Posts: 211
  • Joined: Sep 14, 2020
|
#92859
Hi P.S.
I translated the question stem to mean we're looking for the correct answer to be a Must be False (the 4 incorrect answers are: Could be True). I narrowed down contenders to A, C, E. I see why A is the correct answer choice. Can someone explain why E isn't a must be false answer? Why can't E be the correct answer?

This is the hypothetical I drew and why I chose E:

(In order from 1-3)
W: M, S/O/P, O/P/S
E: ___, U, G
I thought it was a Must Be False answer because per Rule 1, S must go in West (so it can't be 1st in East). And O nor P can go at the same time as M (neither can go 1st in East). Can someone give me a scenario where E is possible?

Also, I want to make sure I eliminated C for the right reasons. Here is the hypothetical I drew to prove C is a Could Be True answer:
(In Order from 1-3)
W: S, O/P, G/P/O
E: M, U, G/O/P
As long as G goes 3rd in either West or East

Can someone please explain?
Thanks in advance
 Robert Carroll
PowerScore Staff
  • PowerScore Staff
  • Posts: 1787
  • Joined: Dec 06, 2013
|
#92875
GGIBA003,

Yes, the correct answer Must Be False. We normally try to keep to phrases using the word "true", so you could also say that we're looking for the one answer that Cannot Be True.

For answer choice (E), there was no reason to have M specifically in the west theater. So just do this:

West: S O P
East: M U G

In fact, that's one of the hypotheticals you drew for answer choice (C), so be more careful about that.

For answer choice (C), your hypotheticals should be more determinate, because G's being at 3 in the East theater is the local condition - it shouldn't be able to vary. Further, the hypothetical I drew above for answer choice (E), which is also compatible with your hypotheticals for answer choice (C), shows that S can be at 1 in the West theater.

Robert Carroll
 g_lawyered
  • Posts: 211
  • Joined: Sep 14, 2020
|
#92922
I see where I went wrong now. I didn't realize that hypothetical can eliminate both answer C and E. Is this a hypothetical that should have been a template? I didn't draw out templates for this game...
Thanks for clarifying that!
 Adam Tyson
PowerScore Staff
  • PowerScore Staff
  • Posts: 5153
  • Joined: Apr 14, 2011
|
#92998
I didn't do templates for this game either, GGIBA003@FIU.EDU, and it's okay if you didn't. But for this question I drew two local diagrams, both with G at 3 in the East per the question stem. In one of them I placed U at 1 in the East and in the other I placed U at 2 in the East. From there I could see every possible solution, and it was clear that answer A was impossible! With G at 3 in either theater, at least one of O or P must be at 2, and that forces M to be at 1, regardless of which theater M or G is in. M cannot be at 2 when G is at 3!
 g_lawyered
  • Posts: 211
  • Joined: Sep 14, 2020
|
#93058
Hi Adam,
The fact that you didn't do templates for this game either does make me feel better about it. Lol
I just took a long time drawing out templates for this many Local Questions , so I thought I missed out by not trying to draw out templates during the game set up. Since I did this PT as full timed test, I did some questions and guessed on others to have enough time for the other games,
Any tips on how to determine the use of Templates per game faster? My rule of thumb is usually, if I don't see big inferences in the set up and see a lot of Local Questions, then I don't spend time attempting to make templates. My problem is the speed at which I do questions (have to skip and guess on some so I can get to the rest of the games). Any tips on getting faster?
Thanks in advance!
 Robert Carroll
PowerScore Staff
  • PowerScore Staff
  • Posts: 1787
  • Joined: Dec 06, 2013
|
#93271
GGIBA003,

The topic of when to do templates is a big one, and we have some resources on that:

viewtopic.php?p=92520#p92520

https://www.powerscore.com/lsat/podcast/21/

It sounds like you're thinking along the right lines. I want to reemphasize, as I say at the first link I'm posting here, that any game where PowerScore would identify templates provides good practice for how to decide on templates.

Robert Carroll
 g_lawyered
  • Posts: 211
  • Joined: Sep 14, 2020
|
#93436
Thank you for the resources Robert!
User avatar
 Sufineeah18
  • Posts: 1
  • Joined: Mar 25, 2023
|
#100655
the question 3 says could be true except, but can u see the (d) option, in that how can S can be on 2 on west theatres. Wouldn't that break the rule . im confused . help
User avatar
 Jeff Wren
PowerScore Staff
  • PowerScore Staff
  • Posts: 389
  • Joined: Oct 19, 2022
|
#100677
Hi Sufineeah,

I'm not sure which rule specifically you are referring to, but I think that you may have misread either one of the rules or the answer choice.

The first rule states that "Sales must be in the WEST theater."

Answer D says that "Sales is presented at 2:00 in the WEST theater."

This doesn't violate the first rule or any of the other rules.

Here is a hypothetical solution to Answer D which you can test against all of the rules to see that it works.

East - U, O/P, G (U at 1:00, either O or P at 2:00, G at 3:00)
West - M, S, P/O (M at 1:00, S at 2:00, either P or O at 3:00)

Answer D is possible/could be true, so it is not correct.

Get the most out of your LSAT Prep Plus subscription.

Analyze and track your performance with our Testing and Analytics Package.